What is Torque: Definition and 1000 Discussions

In physics and mechanics, torque is the rotational equivalent of linear force. It is also referred to as the moment, moment of force, rotational force or turning effect, depending on the field of study. The concept originated with the studies by Archimedes of the usage of levers. Just as a linear force is a push or a pull, a torque can be thought of as a twist to an object around a specific axis. Another definition of torque is the product of the magnitude of the force and the perpendicular distance of the line of action of a force from the axis of rotation. The symbol for torque is typically




τ



{\displaystyle {\boldsymbol {\tau }}}
or τ, the lowercase Greek letter tau. When being referred to as moment of force, it is commonly denoted by M.
In three dimensions, the torque is a pseudovector; for point particles, it is given by the cross product of the position vector (distance vector) and the force vector. The magnitude of torque of a rigid body depends on three quantities: the force applied, the lever arm vector connecting the point about which the torque is being measured to the point of force application, and the angle between the force and lever arm vectors. In symbols:





τ

=

r

×

F





{\displaystyle {\boldsymbol {\tau }}=\mathbf {r} \times \mathbf {F} \,\!}




τ
=


r





F


sin

θ




{\displaystyle \tau =\|\mathbf {r} \|\,\|\mathbf {F} \|\sin \theta \,\!}
where





τ



{\displaystyle {\boldsymbol {\tau }}}
is the torque vector and



τ


{\displaystyle \tau }
is the magnitude of the torque,





r



{\displaystyle \mathbf {r} }
is the position vector (a vector from the point about which the torque is being measured to the point where the force is applied),





F



{\displaystyle \mathbf {F} }
is the force vector,




×


{\displaystyle \times }
denotes the cross product, which produces a vector that is perpendicular to both r and F following the right-hand rule,




θ


{\displaystyle \theta }
is the angle between the force vector and the lever arm vector.The SI unit for torque is the newton-metre (N⋅m). For more on the units of torque, see § Units.

View More On Wikipedia.org
  1. F

    What size flywheel do I need to produce 200kg torque or resist a force of 200kg?

    Hello, https://www.fxsolver.com/solve/ I am using this calculator to figure out how large/heavy a flywheel I need to produce torque of 200kg, or that could resist a force of 200kg applied to it. I am not proficient in mathematics, so my terms or what I'm asking might be not phrases correctly...
  2. K

    Torque on Left Foot of Man in Rotating System

    I saw that the solution states that the torque about the center of mass is zero, since the man does not rotate about its center of mass. However, I then thought about taking the torque about the left foot (so the right foot for the man's POV). Hence: $$\tau_{left} = \tau_{0} + \textbf{R}\times...
  3. A

    Understanding Torque: Applying the Right Hand Rule in Real Life

    What does right hand rule torque mean in real life? engine crankshaft produce torque,if apply right hand rule to crankshat that will mean that torque is act in crankshaft axiis of roatation,which mean axials berings will have huge load on it,,that is not the case in reality,axail bering don't...
  4. ZdMh

    Transmitting Torque: Meaning & Why Not Power?

    When we talk about for ex. a driver rotates a pump, we say "a torque is transmitted to the pump". What does exactly this mean "Transmitting torque"? and why we said Transmitting torque and not transmitting power for ex.?
  5. W

    Power/Torque of Large Hauler, Tank & Bulldozer: Comparing Trees

    I was trying to do some research to find the difference in power of different vehicles. Mainly the difference between a Large hauling semi truck A battle tank and a bulldozer like a d11t. Let's say I hooked up a chain to the back of each of these vehicles and rapped it around a large 3 foot...
  6. PhysicoRaj

    Reaction torque in a traction system

    Hello PF, I am building a traction assembly for a self balancing bot project and I am having some conflict with my intuition and practical testing results. The setup consists of a motor mounted to a chassis, the shaft coupled to a wheel that rests on a surface with non-zero friction. Below...
  7. Y

    Fastener Theory - Determining the Ideal Torque for a Bolt

    So this is for my car. If I have a bolt that I don't know what the ideal torque is supposed to be because the manufacturer of the car doesn't publish a torque, it's possible to calculate it? This can be done just give the specification of the bolt? Regardless of what the bolt is fastening? I'm...
  8. L

    Dual-Axis Solar Tracker Torque requirements

    Hello, I am building a dual-axis solar tracker for educational purposes and I need to determine the required torque to move the solar panel in both the azimuth and elevation directions in order to determine what size motors to use. I have attached a simple sketch illustrating the mechanical...
  9. S

    If two gears are connected by a shaft do they have the same torque?

    1 axle. 2 gears on the same axle fixed in place. Does the larger gear with 48 teeth have the same amount of (force/strength) as the smaller gear of 12 teeth. IE: if I attached a chain to the 12 tooth gear and it could lift a max load of 10 kg, would the 48 tooth gear be able to lift 10 kg as...
  10. P

    Automotive Vehicle Parking Brake Test on a Slope [How Convert to a Flat Ground test?]

    Vehicle Mass : 3 tonnes Manufacturer's instruction is to test vehicle's parking brake effectiveness is to engage parking brake on 30.96 degrees slope. If it holds, it is effective. I do not have 30.96 degrees slope & I want to replicate the same test on flat ground. I was thinking of...
  11. I

    When calculating a car's acceleration, is it ok to substitute Torque?

    1. When calculating a car's acceleration, is it ok to substitute Torque into a= F/ma= F/m T= FrF= T/rwhere a= acceleration, F= force, m= mass, T= Torque, r= radius,a= T/rm ?
  12. LCSphysicist

    Why when the axis is pivoted, there is no torque in its direction?

    I want to say [the bodies is under gravity field]: There cannot be torque alonge AB "since it is pivoted" There cannot be torque alonge AB "since it is pivoted" i think i am missing something.
  13. Y

    Moment of inertia where mass and torque are at a different positions

    The formula for moment of inertia is: I=mr^2 A common derivation for this is: 1. F=ma 2. τ=rma 3. τ=rmrα = r^2 mα This is a rotational version of Newton’s second law, where torque replaces force, moment of inertia replaces mass, and angular acceleration replaces tangential acceleration...
  14. C

    Calculating Force and Torque for Linear motion (image provided)

    This is how I understand the torque (T1) created at the axis of rotation due to the spring would be calculated. This is considering the spring is starting from a fully extended state and is not already compressed. This is considering that (A) has a distance of 2 in. and the force from the spring...
  15. cupcake_rainbow

    Torque Balancing and Virtual Work: Solving for Equations in Force Balancing

    So I tried using force balancing. I have attached files of my solution in my notebook Torque balancing about O which gives me equation 1 And I used property of triangle, for equation 2 But i can't seem to get the right answer Also, I was wondering if it could be done by concept of virtual work.
  16. R

    Determining moment (or torque) around a certain point on a wrench

    Here's a diagram of the problem. There are 3 possible solutions: 163 in-lb clockwise, 163 in-lb counterclockwise, 82.7 clockwise or none of them. After attempting the problem the closest I got to was 82.7 but that was wrong so I'm still stuck.
  17. T

    How to ensure maximum possible joint torque for an actuator?

    Below, I have a joint system that consists of a fixed anchor and a moving arm: What I want is simple: To attach a linear actuator on the moving arm in such a way that the mechanical advantage of the actuator is exactly 1 (Ie the actuator only has to generate exactly the amount of force as the...
  18. Leo Liu

    Torque on a circular current loop

    Can anyone please tell me why the torque on a circular current loop can be calculated by the equation below? In other words, what is the intuition for the formula? Thank you. (I modified my question to a particular case) $$ \vec{\tau} = I \vec{A} \times \vec{B} $$
  19. G

    What is the difference between static friction and rolling resistance?

    Hi, it's probably a frequent question, but I can't seem to wrap my hear around. I'm trying to understand how I reach that value, not just how to get the value. I'm designing a vehicle with 4 wheels. It'll climb a surface with a slope theta (in degrees). This is the FBD that I've come up with of...
  20. Adesh

    What force will be felt by ##B## when a rod is rotated?

    We have a rod ##AB## of mass ##m##, a force (perpendicular to AB) is applied at ##A##. I want to know how much force will ##B## going to feel? When ##F_1## is applied at ##A## rod will rotate about its COM (which lies at the Center) and hence the point ##B## will also move (a little downwards...
  21. PCarson85

    Effect of thrust force on torque?

    If I have a motor that is turning a shaft with only a small torque required but the shaft is also being forced into the motor, axially at a large force, what is the effect on the motor here? If Faxial is large, does this effect T and how does this effect the motor?
  22. D

    Torque on Two Different-Sign Charged Objects

    Hi I am confused in this question. What Can I do after the steps in second photo?
  23. R

    Understanding Kinetic Energy, Angular Momentum & Torque

    Well I am pretty sure that the kinetic energy stays the same because in this case the velocity vector and energy make a ninety degree angle so no work is done, but I am lost about angular momentum. It could decrease maybe if the torque is clockwise while the ship is going in a counterclockwise...
  24. Shubol3D

    Calculate the output power for da Vinci's tank mechanism

    Hello everyone. I'm working on 3D animation about da Vinci tank and why it doesn't work. Please help me find out how to compute: 1: how many men need to move this machine 2: what power on wheel can produce different number of man, like 1, 2, 4, 8.. vehicle mass = 8t men strength = 60kg lever...
  25. D

    Starting Torque of Synchronous Motors

    It is pretty well known among electrical machines folk that large, salient pole electrical machines start typically with the field winding shorted and starting torque developed by current induced in the damper windings, acting much like a squirrel cage induction machine. The net machine torque...
  26. TURNIGY

    Trying to find the torque required to turn the crankshaft in my mechanism

    Hello!, First post and I am excited. I am an artist and want to see if the motor I have selected for this mechanism will work for a project I am working on. I am trying to see if the gearbox/motor I have sourced for this mechanism is powerful enough to drive it. The weight of the object I...
  27. D

    Torque calculations for a small electric vehicle

    Hello guys, I'm trying to calculate the torque for a 3 wheeler electric (small vehicle). 2 driving wheels and one driven wheel. At the driving wheels we have a BLDC motor for each wheel. We want to calculate the torque for resting( not going down hill) on inclined plane (angle={5,10,15...
  28. Z

    Increasing the output torque of a DC motor with gears

    Hello everyone, I see videos showing the output torque can be increased by using gears and this is usually demonstrated with DC motors (simple DC motors can be removed from toys). I understand the general logic: * The DC rotates in a high speed. * The gear system reduces the output speed. * If...
  29. B

    Torque on a rectangular coil in a uniform magnetic field

    So this was a section taken out from a question which I am trying to do shown below I have drawn a sketch to help me visualise of what is going on I have used Fleming's left hand rule to help me determine what direction the force is facing on each side of the coil. For the last part in...
  30. D

    Is my elbow the point of application of force of a torque?

    if i pick up a bucket i apply torque to the bucket. my shoulder is the fulcrum. is my elbow the point of application of force? just trying to understand the physics?
  31. B

    A rotating system of two point particles with inner torque

    Lets say we have a system of two point particles (1. and 2.) which are rotating around an axis. What is written next in my physics course book is: The torque of a 2.body on the 1. body is M21=r1xF21 and the torque of the 1.body on the 2.body is M12=r2xF12. Understandable. But how? There is no...
  32. A

    Automotive Motorcycle: Flywheel mass and rear wheel torque

    Hello, I made this account to ask this question that I’ve been loosing sleep over. My question is say you have 2 identical motorcycles the engine the exact same and everything the only difference is the mass of the flywheel one bikes engine has a flywheel twice the mass as the other will the...
  33. L

    Integral for the calculation of torque

    Hello, I found an integral to calculate the torque from the applied torsional shear stress, and I didn't find an explanation of how this integral is deviated. Where does it come from? Could someone explain? T = ∫τ⋅r⋅dA = ∫τ⋅2πr⋅dr, where T is the torque and τ the shear stress. Thanks a lot!
  34. ari-anne

    Finding the center of gravity with torque

    I don't know how to find the center of gravity much less in this problem, so I just created a diagram of the pole with all the forces.
  35. Saptarshi Sarkar

    Torque on a rotating system system

    I calculated the total moment of inertia of the system to be ##2ml²Sin²θ##, so the angular momentum is ##2ml²ωSin²θ##. To get the torque on the system I need to differentiate the angular momentum but I don't have any time dependent terms. What should I do?
  36. T

    Why is the gear ratio reversed in a planetary gear system?

    Hello, I am confused about something. I will take the time to work it out, but right now, I am using it to remember. And I am now confused. Go here: https://www.smlease.com/entries/mechanism/gear-train-gear-ratio-torque-and-speed-calculation/ Scroll down to the section on...
  37. O

    Analytical mathematical modeling of the torque caused by the laundry

    Hello my dear physicists, I'm trying to model the varied generated (needed)Torque to rotate a washing machine Drum during a Washing Process so i assumed that the Model has as Input the target vilocity and as an Output the new needed torque to rotate the Drum(to be as a input for the motor...
  38. R

    Can the Normal Force Produce a Torque on an Object on an Incline?

    Hi everyone I can't understand how normal force produce torque isn't normal force acts normally on the plane so the perpendicular distance in case if it was like block will be zero, so normal force doesn't produce torque. "case of box"
  39. cs44167

    Torque on a Beam Due to Gravity

    I tried using r * f * sin theta and calculated this: 1.14 m * 9.80 m/s/s * 7.78 kg = 109.7 N*m this was wrong; I needed three significant figures so I did 1.10E2 N*m which was also wrong. Since the torque is due to gravity; would it be -1.10E2 N*m since it’s angle is -270° which is -1?
  40. S

    Find the Right Motor for a Vertical Load Torque

    If I'm to choose a motor to suit a vertical load (torque)-see the attached picture for more illustration-. What is the criteria should i follow? After i have done the math for the weight of the object to be rotated, the torque to rotate the load is to be horizontal, and i only have the weight...
  41. R

    Pulleys with Torque: Free Body Diagram Analysis

    Free body diagram is below, but I have confusion about the tension. The tension on the block and pulley should always be the same by Newton third law. so T1=T1' and T2=T2'. Part B) The system is in equilibrium so net torque on each pulley is equal to zero. Therefore T1'=T2' and T2'=T3. Applying...
  42. A

    Opposing force of the axle when a torque is applied to a wheel?

    Okay let's say we have a wheel attached to a fixed axle, free to spin, ignore friction and we apply a force on the edge of the wheel. That force creates a torque, does the axle have an opposing force to keep the wheel/axle assembly from accelerating linearly? Ive learned on this forum that when...
  43. O

    The torque (and stalling torque) of a servo motor

    I'm using a servo to lift a pen. The pen and a bit of mechanism weights 35g. The servo I use to bring the pen up and down claims to have a torsional moment of 1.5kg/cm. Sometimes they call it torque in the specs. But surely that does not mean they are claiming that if I hung a 1.5kg weight a 1...
  44. hugo_faurand

    [Mechanics] Calculate the braking torque

    Hello everyone ! I've got a problem in engineering class with a braking system (picture linked). In the first part I calculated the friction force $$ \vec{B} $$ with components : T (following y axis) : 2073.6N N (following x axis) : 5760N. For the first question I struggle a lot because I...
  45. B

    Confusing Derivation of torque acting on disk question

    Not sure how they obtained an answer of (2meukenetic *Mg/R^2)*r^2 dr = dt When they say disk I am assuming it's not ring shaped. So since F * Dr= DT where I guess F acting on a small point on the disc is constant from the table top. And Dr is the variable distance from a random small point from...
  46. kolleamm

    Calculating Torque needed to raise an object

    I'm trying to find out if my motor can rotate a robotic arm upwards from a resting position (we can assume the arm is a cylinder for simplicity). The cylinder has an outer diameter of 5.6 cm , and an inner diameter of 5.0cm Therefore its cross sectional area in that case should be pi(2.8)^2 -...
  47. D

    Finding the torque of a robotic arm

    let's say we have a robotic arm, connected to a servo motor. the servo motor is rated at 500 kilogram-centimeter. the robotic arm lifts something upwards. will the torque remain constant? if so, torque equals force times distance to point of application of force time sine angle, will the angle...
Back
Top